This is a must be true question, indicated by the question stem: Which one of the following conclusions can be validly drawn from the passage?

Our stimulus opens with the conditional indicator only, which indicates necessity. We learn that for someone to understand Patrick’s irrational behavior it’s required that they be an expert in some branch of psychology. Unfortunately for Patrick, the next sentence, beginning with the conditional indicator no, informs us that if you’re an expert, then you won’t be certain of your ability to solve someone else’s problem. So the only people who will understand Patrick’s problem will be people who won’t be certain about their ability to solve it, right? Wrong. An important inference we should make is that a key detail in that second conditional is that it only applies to someone else’s problem. We need to remember that for all we know Patrick himself could be an expert in some branch of psychology, in which case it would be entirely possible that he understand his own behavioral issues and is certain that he can solve them. Our final sentence tells us that Patrick wants to solve his problems; interesting, but want doesn’t tell us much beyond Patrick’s desire. And that’s all we get! This is a 5 star question, and it is easy to see why. Let’s take a look at the answers:

Answer Choice (A) As always on a must be true question, we should be judging answer choices based on whether they could be false. As noted in our breakdown of the stimulus, we don’t know whether Patrick is an expert in some branch of psychology, so this answer could be false.

Answer Choice (B) Same issue as A but more explicit; we have been given no information about Patrick except that he (i) has a behavioral problem, and (ii) wants to solve it.

Answer Choice (C) Answers A to C all depend on you failing to recognize that we don’t know whether Patrick is an expert. But even if he was, our experts being uncertain rule only applies with reference to someone else, so this still could be false.

Answer Choice (D) We have been told that you need to be an expert in psychology to understand behavioral problems, but we haven’t been told this understanding is required to offer solutions.

Correct Answer Choice (E) Since we know that experts are never certain about solving other’s problems, the only way Charles could be certain is if he wasn’t an expert and therefore couldn’t understand Patrick’s problems.


Comment on this

This is a sufficient assumption question, as the question stem asks: Which one of the following is an assumption that would permit the conclusion above to be properly drawn?

We’re told that every photo must in some ways be true - that stuff in between the commas is science stuff that basically means that because the light of what we capture hits the film. The next sentence begins with a “but” which indicates a potential pivot; the argument goes on to say because of things like Photoshop or angles/posing (cue social media) it doesn’t show the whole trust and is false. Our conclusion comes in and says nothing can ever be proven with photos. First, “nothing” is very strong. Second, being false and proving something are two different, albeit related, ideas. What if you can prove something to be false with an altered photograph by comparing it to what’s actually the case? Let’s link these ideas up with a rule: “If a photograph can be altered to prevent showing the whole truth and is therefore false, then nothing can be proven with it.”

Correct Answer Choice (A) This is correct because it links up our premises with the conclusion and forced our conclusion to be true. While it’s not a perfect paraphrase of our rule, it conveys the same thing.

Answer Choice (B) We’re told that photographs cannot express the whole truth. What does knowing the whole truth have to do with our argument? With our premises and this answer, we cannot force the conclusion.

Answer Choice (C) Being able to figure out whether or not a photograph is truthful does not help push out our conclusion; we still won’t know what is true or false, and this answer choice does not bridge the gap between something being false and figuring out what is not provable.

Answer Choice (D) This does not help justify that nothing can be proven with a photograph. The answer choice adds more information about finding out the truth of the scene of the photograph and then determining what we can use to photograph as evidence. This is more information unrelated to justifying our conclusion.

Answer Choice (E) This would weaken our argument - this is out.


Comment on this

This page shows a recording of a live class. We're working hard to create our standard, concise explanation videos for the questions in this PrepTest. Thank you for your patience!

We know this is a sufficient assumption question because of the question stem: “which one… if assumed… would allow [the author] to draw her properly to draw her conclusion…” Interestingly, the conclusion is given to you in the stem. This should help in our formal analysis to figure out what the conclusion is.

Sufficient assumption questions tend to be very formal. We’re looking for a rule that would validate the conclusion, specifically by bridging the premise and conclusion through the rule. Not only are we extrapolating the rule from our argument, but we’re also using that rule to render the argument “valid.” The way to prephrase our answer choice is by tying our premises and conclusion together into a rule: “If [premise] → then [conclusion].”

Our first sentence tells us that an antitheft alarm may stop an attempted theft at night on a crowded street. I’m imagining an incredibly loud siren going off in a middle of a downtown area... I think it’s possible that theft could be prevented with a loud alarm like that.

The next sentence gives an alternative cause for the alarm going off: instead of a thief, it could be a branch or another form of contact.

The third sentence puts aside the causes and says in any of the situations in which the alarm goes off at night, it’s disturbing the sleep of people in the neighborhood. Makes sense - a blaring alarm would definitely disturb my sleep!

Our last sentence, the conclusion which we read in our stem, is: out of consideration for our neighbors, we should deactivate the alarms when parking in crowded city neighborhoods at night.

Why should they?? Protecting a car is probably more important and valuable to its owner than someone else’s sleep, and there doesn’t seem to be a moral code that says people should deactivate their cars if it bothers people’s sleep. On the chance that the car is getting stolen, an alarm would be the best thing to help avoid losing the car! This prescriptive conclusion is assuming that others’ sleep is more important than protecting someone’s car.

What we need is a rule that bridges our premises to our conclusion and validates this prescription. Our premise here is that if the alarm goes off, then people’s sleep is disturbed, and we should be considerate of this. Our conclusion is that owners of these cars should deactivate their alarms. Put them together to make our rule! If people’s sleep is getting disturbed and we should be considerate of this → owners should deactivate car alarms at night.

Answer Choice (A) This would weaken the argument! This is putting the protection of peoples’ cars over peoples’ sleep.

Answer Choice (B) In most cases? What about some cases in which it’s actually a theft? And besides this, the answer choice still does not justify why we should deactivate our alarms. Just because it is a false alarm, it does not validate the argument.

Correct Answer Choice (C) While it’s not a perfect paraphrase of our rule, it gets to the idea that sleep is more important than protecting cars.

Answer Choice (D) Remember, our conclusion is prescriptive. How does saying “people who have alarms are inconsiderate” help our argument? Does this mean they should deactivate their alarms? No – this is out.

Answer Choice (E) We don’t really care about what happens to the alarms during the day, we’re concerned with why people should turn alarms off at night. This is supplemental information that does nothing for our stimulus.


Comment on this

This page shows a recording of a live class. We're working hard to create our standard, concise explanation videos for the questions in this PrepTest. Thank you for your patience!

This is a Sufficient Assumption (SA) question and we know this because of the question stem: “conclusion is properly drawn from the premises given if which one of the following is true...”

Sufficient assumption questions tend to be very formal. We’re looking for a rule that would validate the conclusion, specifically by bridging the premise and conclusion through the rule. Not only are we extrapolating the rule from our argument, but we’re also using that rule to render the argument “valid.” The way to prephrase our answer choice is by tying our premises and conclusion together into a rule: “If [premise] → then [conclusion].” Sometimes, a rule is not always helpful. Recognition of what the gap is might be a more useful prephrase.

The first sentence is a comparison between the pull stroke between an S-shaped rake and a straight-handle rake. The S-shaped rake will reduce compression on the spine to 1/5 of what is it with the latter.

The next sentence starts with a “however” which makes me think there is some information that is different or runs contrary to the information presented before in some way (pivoting). Reading on, we learn that on the push stroke, the s-shaped rake exerts 5x more pressure than the straight rake.

In the next sentence, we learned that the compression in the pull/push for the straight rake is not dangerous, but it is above the danger lever for the s-shaped rake. So far all of this is just information/premises. The next sentence begins with “therefore,” and states that straight rakes are better than s-shaped rakes for minimizing the risk of spinal injury. This is our conclusion.

What is the gap here? Well, jumping from compression stress injuries on the spine to all injuries on the spine. And also, the statistics of risk are equal: X being 5 times Y and Y being 1/5 of X is equal. Why should we prefer a straight-handled rake if the risk of injury for pulling in one and pushing in the other is the same? Our rule needs to relate stress caused by pushing (because that’s the one where s-shaped rakes are worse) to factors associated with all spinal injuries when raking. Not only that, but it needs to explicitly say that the two go hand in hand, or compression related to the push (and not pull) stroke is one of the most important factors in all spinal injuries caused by raking.

Correct Answer Choice (A) This answer choice is relating compression from pushing and spinal injuries in a causal relationship by saying that compression stress from pushing rakes is the only cause of spinal injuries from raking. So, of all the injuries that happen with raking, spinal injuries are only caused by compression stress from pushing the rake, which means that we can conclude that straight rakes are better for minimizing spinal injuries in general.

Answer Choice (B) This is not the correct answer. It’s not relevant to making our conclusion valid – sure, a gardener can still get spinal injuries by using even a straight-handled rake. If we plug this back into our stimulus, it doesn’t really do anything.

Answer Choice (C) This is describing what is happening with the redesign’s flaw. If this was a principle question, this would be a great answer choice. However, this is an SA question and our conclusion has nothing to do with a redesign having no gain in efficiency. We’re making a judgment claim on which rake is better as it relates to a specific category of injuries.

Answer Choice (D) This is basically saying that some strokes have to be pull strokes, which means that there is some risk of compression injury with a straight handle. This attempts to weaken our answer choice but fails because some risk while raking is compatible with our conclusion (it’s about lowering risk, not eliminating it).

Answer Choice (E) This is pointless to our argument. We’re trying to say that one is better than the other, not introduce a third option.


Comment on this

This is a most strongly supported question, as the question stem asks: If both Monroe’s conclusion and the evidence on which he bases it are correct, they would provide the strongest support for which one of the following?

From the question stem alone we already know someone named Monroe is going to be giving us a conclusion supported by some evidence. The first sentence gives us a phenomenon; Monroe ate at the Tip-Top Restaurant and enjoyed the meals, but afterwards became ill each time. He must have really enjoyed it to go back after being sick two times in a row! Since we have started with a phenomenon, Monroe getting sick every time he eats at Tip-Top, we should expect a causal hypothesis as to why this correlation is occurring. The next sentence gives us some more detail about this correlation. Monroe ate three different meals at Tip-Top, but each one had hot peppers. So this Monroe getting sick every time he eats at Tip-Top correlation is also more specifically a Monroe getting sick every time he eats hot peppers correlation. Where the first sentence probably made you think, “Maybe you should try a new restaurant, Monroe!”, this new information has properly altered your hypothesis to, “Maybe ask them to skip the hot peppers next time, Monroe!”. The final sentence tells us that Monroe agrees and gives us his conclusion, his hypothesis for why he kept getting sick; it was the hot peppers causing him to feel sick after all three meals. He probably should have been able to figure that out after two meals, but good for Monroe!

Having now read the stimulus, and remembering that we are supposed to assume Monroe’s hot pepper hypothesis is correct, our job is to select the answer that is most strongly supported based off of it. Let’s take a look at the answer choices:

Answer Choice (A) What this answer choice does is confuse sufficiency with necessity. Just because hot peppers are enough for Monroe to feel sick, doesn’t mean they are required for him to be sick. Maybe he’s also allergic to pine nuts and the all-you-can-eat specials includes a really nice Pesto pasta! If you chose this answer, you might want to review conditional logic.

Correct Answer Choice (B) This is the correct answer because it does what A tries to do without its error of confusing enough with required. Where A said Monroe can eat anything without hot peppers and not get sick, B correctly infers that if Monroe ate a different dish with hot peppers he would have still gotten sick, because hot peppers are enough for him to be ill.

Answer Choice (C) Same issue as A, this question incorrectly assumes that hot peppers are the only possible cause of Monroe feeling ill.

Answer Choice (D) We need to assume that we just don’t know about Monroe’s eating history for this answer to follow from our stimulus.

Answer Choice (E) This should be clearly wrong. We know nothing about Monroe’s restaurant history outside of Tip-Top. Maybe Monroe is really bad at noticing when an ingredient makes him feel sick and he’s actually eaten hot peppers 20 times at other restaurants and gotten sick, and only now after three meals at Tip-Top made the connection.


1 comment

This is a most strongly supported question, since the question stem says: Which one of the following conclusions is most strongly supported by the results of the experiments?

As you should have noticed in the question stem, this stimulus involves experiments. We should therefore be on the lookout for a phenomenon. The first sentence begins with the conditional indicator only, and from it we learn that some strains of tobacco are resistant to tobacco mosaic virus; important to note that resistant here doesn’t mean they resist infection, but rather they resist developing symptoms and becoming diseased. This resistance is our phenomenon. The first of the experiments gives us a correlation. Tobacco plants infected with the mosaic virus who resisted its effects had increased levels of salicylic acid, while non-resistant plants had no increase in their levels. Resistance to the virus correlates with more salicylic acid. The second experiment greatly strengthens the correlation by artificially raising the salicylic levels of half of 50 non-resistant plant, with every high salicylic level plant resisting the disease and every normal plan succumbing to it.

If we were to summarize this stimulus, it is essentially: phenomenon, correlation, even stronger correlation. What we never receive in this stimulus is a conclusion, or more specifically in this case, a hypothesis. If we pre-phrase, we would expect a conclusion to this information to infer causation from correlation, and conclude that the increased salicylic acid at least partly causes the mosaic resistance. We should be on the lookout for an answer that introduces this hypothesis, while eliminating any answers that have little to no support in the stimulus. Let’s take a look at our options:

Answer Choice (A) None of the experiments involved salicylic acid removing symptoms, but only preventing them. Our correlation is between salicylic acid and resistance, not healing.

Correct Answer Choice (B) This answer does exactly what we should have predicted in our pre-phrasing; it introduces the expected hypothesis that salicylic acid at least partly explains why some plants are able to resist the mosaic virus.

Answer Choice (C) This information is not only not supported, but likely false based on the stimulus. The salicylic acid is described as naturally occurring in the plants, and the difference between the resistant and non-resistant plants is whether there was an increase in salicylic acid.

Answer Choice (D) Remember we are told about an increase in the salicylic levels following infection; if the plant is uninfected the stimulus suggests that its salicylic acid levels will be normal, and maybe even indistinguishable from those of non-resistant plants.

Answer Choice (E) This answer might be appealing since it does emphasize the correlation between an increase in salicylic acid and resistance to the virus. The problem is that the experiments involved artificially injecting plants to raise their salicylic levels, and we have no information about whether it is possible increase the plant’s production of the acid.


Comment on this

This page shows a recording of a live class. We're working hard to create our standard, concise explanation videos for the questions in this PrepTest. Thank you for your patience!

This is a weakening question, since we are asked: Which one of the following, if true, most seriously weakens the argument?

This stimulus is relatively straightforward. We are first told that a portion of economically useful raw materials are both nonrenewable and in limited supply on Earth. So we can’t make more of these resources and we don’t have such a big supply that we would never run out. Following this premise we get a sentence beginning with the conclusion indicator therefore and the conditional indicator unless. If we translate this conditional conclusion, we should end up with something along the lines of: no materials can be obtained outside earth → unable to accomplish what is now accomplished with those materials (referential phrase: the useful, non-renewable, and limited supply materials).

When we are faced with a weakening question involving a standard argument, we should always consider if the argument relies on a large assumption. In this case, we are concluding that if we can’t find more of these materials, we won’t be able to do what we use them for right now. The major assumption here is that these materials are required for what we do with them. For example, just because we currently use gas to produce electricity doesn’t mean that gas is required to produce electricity. An easy way to weaken this argument is to explicitly state that these materials aren’t required, and could be replaced with alternatives. We should be on the lookout for an answer choice that does this. Let’s see what we get:

Answer Choice (A) The argument we are trying to weaken is about the portion of economically useful materials which aren’t renewable. Even if there are renewable resources, unless they are viable alternatives to our non-renewables, we don’t care about them.

Answer Choice (B) This answer choice just suggests that the conditional conclusion of our argument will be triggered, since it is hard to get resources outside of earth. This does nothing to weaken or even strengthen the argument.

Correct Answer Choice (C) This contradicts the assumption we identified in the stimulus. If these limited nonrenewable resources can be replaced with alternatives, then even if we can’t find more of them we can still keep doing what we are doing with them. In short, this says the resources aren’t required, which is exactly what the argument assumes they are.

Answer Choice (D) Who cares? The argument we are trying to weaken has a conditional conclusion; it makes a prediction about what will happen in a particular case. The worthiness of what the resources are being used for is irrelevant to whether it will be impossible to continue should they run out.

Answer Choice (E) This answer fills in one of the assumptions the argument makes; that we will ever run out of these resources just because there is a limited supply of them. For that reason, if anything, all this answer does is strengthen what we want to weaken.


Comment on this

This page shows a recording of a live class. We're working hard to create our standard, concise explanation videos for the questions in this PrepTest. Thank you for your patience!

We can identify this question as Method of Reasoning because of the question stem: “In countering the original conclusion the reasoning above uses which one of the following techniques?”

When dealing with a Method of Reasoning question, we know we are looking for an answer choice that correctly describes the structure of our entire argument. Our correct answer is going to fit the argument exactly. Our wrong answer choices likely explain argument structures we are familiar with, but that simply don’t apply to the specific question we are looking at. Knowing what the right and wrong answers are going to do, we can jump into the stimulus.

The argument begins by telling us about a correlation; those who play bridge tend to have better short-term memories than those who do not play bridge. We know right away that the presence of two qualities at the same time does not mean they caused each other. The argument proceeds by affirming this exactly. We are told that although this correlation was previously concluded to indicate a causal relationship meaning bridge causes better short-term memory, it is just as likely that having a better short-term memory makes the game of bridge more intriguing to that particular group of people.

Ultimately, our stimulus outlines an incorrect conclusion on the basis of a correlation and then goes on to explain another possible interpretation from the evidence. Knowing this, we can jump into answer choice elimination.

Answer Choice (A) If our stimulus were challenging the representativeness of the study included, we would expect the argument to bring up the number of people involved in these different groups or how well these groups represent the rest of the population. Without this information we can eliminate answer choice A.

Answer Choice (B) This answer accuses our stimulus of drawing a conclusion about what is considered “appropriate therapy.” Without this emphasis in the text we can eliminate B.

Answer Choice (C) Our stimulus does not depend on some sort of misunderstanding of the facts involved in the scenario. Thus, we can nix answer choice C.

Correct Answer Choice (D) This is exactly what we are looking for! This is the only answer choice that identifies the alternative possible explanations for the facts presented in the argument.

Answer Choice (E) This answer choice is tricky. But our stimulus does not go quite far enough to say it is describing a flaw of the previous conclusion. Because our stimulus only points out the existence of another possible conclusion we cannot confirm answer choice E.


Comment on this